A sequence can be generated by using fn = 2f(n-1)+1, where f1 = 4 and n is a whole number greater than 1. What are the first four terms in the sequence?

Answers

Answer 1

Answer:

{9,19,39,79}

Step-by-step explanation:

Recursive Sequences

The recursive sequence can be identified because each term is given as a function of one or more of the previous terms. Being n an integer greater than 1, then:

f(n) = 2f(n-1)+1

f(1) = 4

To find the first four terms of the sequence, we set n to the values {2,3,4,5}

For n=2

f(2) = 2f(1)+1

Since f(1)=4:

f(2) = 2*4+1

f(2) = 9

For n=3

f(3) = 2f(2)+1

Since f(2)=9:

f(3) = 2*9+1

f(3) = 19

For n=4

f(4) = 2f(3)+1

Since f(3)=19:

f(4) = 2*19+1

f(4) = 39

For n=5

f(5) = 2f(4)+1

Since f(4)=39:

f(5) = 2*39+1

f(5) = 79


Related Questions

WILL GIVE BRAINLIEST PLS HELP
Solve for y in terms of w, x, and z.
x=zyw
y=

Answers

Answer:

y= x/zw

Step-by-step explanation:

Isolate the variable by dividing each side by the factors that dont contain the variable

x=zyw /zw

y=x/zw

cost of an oil change: $14.50
mark up 50% ​

Answers

Answer:

$21.75

Step-by-step explanation:

Convert 50% markup into decimal

1.5

then calculate

14.50 × 1.5

21.75

Cost of oil change with markup is $21.75

Hope this helps!

Have a nice day!

If you find this helpful

Pls consider marking my answer as Brainliest! It would mean a lot!

PLS HELP ASAP WILL GIVE BRAINLIEST CONVERSIONS
When buying snacks, you want to get the most amount of candy for your money. Which candy
should you buy?
Sour straws: $2.60 for 8 oz
Assorted chocolates: $3.15 per pound
Jellybeans: $1.56 for 200 grams

Answers

Answer: Assorted chocolates: $3.15 per pound

Step-by-step explanation:

Sour straws: $2.60 for 8 oz  = $5.20 per pound

Assorted chocolates: $3.15 per pound

Jellybeans: $1.56 for 200 grams = $3.54 per pound

what is 7/3 + 4/7 ???????

Answers

Answer:

2 19/21, 61/21

Step-by-step explanation:

First, lets make the denimonators, "3 and 7" common. 3 and 7 are prime numbers, so the common denonminator is "21".

For 7/3, multiply by "7/7" and for 4/7 multiply by "3/3".

49/21 + 12/21

Add.

61/21

61 is a prime number so this answer will be your final answer.

(unless you have to put it as a mixed number which would be 2 19/21)

Which statement about the physical change of liquid water boiling into steam is true?

The heat added represents an energy change.
The action cannot be reversed.
The steam cannot conserve mass.
The weight lost represents a mass change.

Answers

Answer:

The heat added represents an energy change.

Answer:

A the heat added represents an energy change trust i got it right

Step-by-step explanation:

what is the slope of a line that is perpendicular to the line y=-1/3×+5


a. -2

b. -1/2

c. 1/2

d. 2​

Answers

Answer:

im guessing you meant -1/2 for the slope of the first line so it is going to be d. 2

Step-by-step explanation:

The slope of a perpendicular line is the negative reciprocal of the other line

what’s is the answer to this ?

Answers

Answer:

162

Step-by-step explanation:

I think this is right because angle one is the same as angel two

18*2=36

360-36=324

324/2=162

Answer:

2= 162*

Step-by-step explanation:

magic

What are the numbers in pi

Answers

Answer:

Well, I don't know all of them but if you really want to know enter it in a calculator and it'll show (hey, I rhymed lol)

pi = 3.141592654....

Hope that helps and have a great day!

3.1415926535897932384626433832795028841971693993751058209749445923078164062862089986280348253421170679821480865132823066470938446095505822317253594081284811174502841027019385211055596446229489549303819644288109756659334461284756482337867831652712019091456485669234603486104543266482133936072602495420647428097356258070669831069799352606933921356858813912148073547284632277849080870024677763036055512323866562951788537196730346347012229395816067925091532174890308408865160611190114984434123501246469280288059961342835118847154497712784733617662850621697787177438243625657117794500644777183702219991066950216567576440449979407650379999548450027106659878136038023141268369057831904607927652972776940436130230517870805082942280202329012216301023097721515694464279098021908266898688342630716092079140851976952355534886577434252775311972474308730436195113961190800302558783876442060850447306312992778889427291897271698905759252446796601897074829609491906487646937027507738664323919190422542902353189233772931667360869962280325571853089192844038050710300647768478632431910002239297852553723755662136447400967605394398382357646069924652600890906241059042154539279044115295803453345002562441010063595300395988644661695956263518780606885137234627079973272331346939714562855426154676506324656766202792452085813477176085216913409465203076733918411475041401689241213198268815686645614853802875393311602322925556189410429953356400957864953409351152664540244187759493169305604486864208627572011723195264050230997745676478384889734643172159806267876718380052476968840849891850861490034324034767426862459523958903585821350064509981782446360873177543788596776729195261112138591947254514003011805034378752776644027626189410175768726804281766238606804778852428874302591452470739505465251353394595987896197789110418902929438185672050709646062635417329446495766126519534957018600154126239622864138977967333290705673769621564981845068422636903678495559700260798679962610190393312637685569687670292953711625280055431007864087289392257145124811357786276649024251619902774710903359333093049483805978566288447874414698414990671237647895822632949046798120899848571635710878311918486302545016209298058292083348136384054217200561219893536693713367333924644161252231969434712064173754912163570085736943973059797097197266666422674311177621764030686813103518991122713397240368870009968629225464650063852886203938005047782769128356033725482557939129852515068299691077542576474883253414121328006267170940090982235296579579978030182824284902214707481111240186076134151503875698309186527806588966823625239378452726345304204188025084423631903833183845505223679923577529291069250432614469501098610888999146585518818735825281643025209392852580779697376208456374821144339881627100317031513344023095263519295886806908213558536801610002137408511544849126858412686958991741491338205784928006982551957402018181056412972508360703568510553317878408290000415525118657794539633175385320921497205266078312602819611648580986845875251299974040927976831766399146553861089375879522149717317281315179329044311218158710235187407572221... and so on

The equation is z=8+6x-px and you're solving for x

Answers

Answer:

p= 6x−z+8 /x

Step-by-step explanation:

Answer:

x= z-8/6-p

it says my answer is to short so i'm going to write a bunch or words

((-4)+(-5)+-1))/3 The / signs is division

Answers

Answer:-1 and 1/3

Step-by-step explanation:-4+-5=-9 -9+-1=-10 -10/3=-1 1/3

Find the midpoint between A (6, 3) and B(-1, 4)

Answers

Answer:

(5/2, 7/5)

Step-by-step explanation:

Answer:

(5/2,7/2)

Step-by-step explanation:

The midpoint formula is

(x1+x2/2),(y1+y2/2)

=(6+(-1)/2), (3+4/2)

=(5/2,7/2)

Michael has $1,058.60 in his checking account. He is going to spend $499.66 on a new television, and he will spend the rest on speakers that cost $42.00 each. Which of the following inequalities would determine the maximum number of speakers, x, Michael can buy without spending more money than he has in his account?
help

Answers

Answer:

he can buy 13 speakers.

Step-by-step explanation:

first you do 1,058.60- 499.66.  Then you take the answer and and divided it by 42.00 and got 13.

Use the distributive property to simplify the following expression-(8+b)

Answers

Do you see the negative sign in front of the parentheses? That is -1.

Multiply each term inside the parentheses by -1.

Doing so, we get -8 -b.

The data set represents the responses, in degrees Fahrenheit, collected to answer the question "How hot is the sidewalk during the school day?". 92, 95, 95, 95, 98, 100, 100, 100, 103, 105, 105, 111, 112, 115, 115, 116, 117, 117, 118, 119, 119, 119, 119, 119, 119 a. Create a dot plot to represent the distribution of the data. b. Create a histogram to represent the distribution of the data. c. Which display gives you a better overall understanding of the data? Explain your reasoning.

Answers

Answer:

a) Please find attached the created dot plot

b) Please find attached the created histogram

c) The dot plot

Step-by-step explanation:

The given data on the responses to the question on "How hot is the side walk, during the school day", is presented as follows;

92, 95, 95, 95, 98, 100, 100, 100, 103, 105, 105, 111, 112, 115, 115, 116, 117, 117, 118, 119, 119, 119, 119, 119, 119

Given that manner of construction of the dot plot and the histogram, the display of the results are similar, however, due to the low frequencies of the data, the dot plot provides a view that allows quicker and clearer understanding of the data

slope (M)= rise/run=y/x=(y2-y1)/(x2-x1)
1a. (1,2) and (7,9)
1b. (5,-1) and (0,3)
1c. (12,5) and (9,8)

Answers

Answer:

1a. 7/6

1b. -4/5

1c. -1

Step-by-step explanation:

1a. 9-2/7-1 = 7/6

1b. 3--1/0-5 = 4/-5 = -4/5

1c. 8-5/9-12 = 3/-3 = -3/3 = -1

Answer:

1a. 7/6

-----------------------

1b. -4/5

-----------------------

1c. -1

Help me ASAP Ill give Brainliest



What is the probability that a jury chosen at random includes at least 10 men? Calculate P(at least 10 men).

Answers

Answer:

.00552

Step-by-step explanation:

To find the probability of choosing a jury with at least 10 men, add the individual probabilities of choosing juries with 10 men, 11 men, and 12 men and then add the individual probabilities (because each case is exclusive).  

Adding the three probabilities, the probability of a jury with at least 10 men is:

P(at least 10 men)

P(10 men) + P(11 men) + P(12 men)

= 0.005 + 0.0005 + 0.00004

= 0.00552

Answer?!?????????????

Answers

Answer:

A i think

Step-by-step explanation:

Answer:

its   c i hope i helped-shelby

Step-by-step explanation:

HURRY, PLEASE HELP!!!

Answers

Answer:

Equilateral, Isosceles, and Scalene triangles all have at least one pair of equal angles.

The angles must have the same degree and most of the time come when connected by or adjoined by 2 equal side lengths.

Hey there people Im doing a poll

Out of these people who r ur favorites?

WilburSoot

TommyInnit

Tubbo

Dream

GeorgeNotFound

Sapnap

Fundy

Skeppy

BBH

Quackity

Niachu

TechnoBlade

Punz

Ponk

aweSAMdude

Answers

my favorite is probably aweSAMdude

Answer:

For me either skeppy or dream...probably skeppy

Step-by-step explanation:

Find the geometric mean of 6 and 9.

Answers

Answer:

the answer is 7.35

Step-by-step explanation:

I’m not sure how to do this I’m post the picture below please help!!!

Answers

[tex]\mathfrak{\huge{\pink{\underline{\underline{AnSwEr:-}}}}}[/tex]

Actually Welcome to the Concept of the Mathematics, this is called as function inside another function, just we have replace the condition of x here,

1)g(f(4)) = here, f(4) = 4-7= -3

so now, g(-3) = (-3)^2 =9

g(f(4)) = 9

2) f(g(4)) = 16-7 = 9

f(g(4))= 9

3) g(f(-1)) = 64

4) f(g(-1)) = -6

done.

Solve the inequality: -7y - 3x < 21

Answers

Answer:

x>  -7/3y -7

Step-by-step explanation: -7y-3x<21

Step 1: Add 7y to both sides.

−3x−7y+7y<21+7y

−3x<7y+21

Step 2: Divide both sides by -3.

-3x/-3 < 7y+21/-3

x>-7/3y -7

Write the quadratic equation whose roots are −6 and 4, and whose leading coefficient is 4. (Use the letter x to represent the variable.)

Answers

Answer:

4(x-4)(x+6)

Standard form: 4x^2+8x-96

Step-by-step explanation:

Since the roots are set to be -6 and 4, we need to find 2 linear equations that are equal to 0 when x is at -6 and when x is at 4, and multiply them together. That would look like this:

(x-4)(x+6)

Notice how if I plug in 4 or -6 for x, the whole thing is going to be 0.

Now, if we were to factor this, the leading coefficient would be 1. Since we want to change the leading coefficient while still preserving our roots, we can just multiply the whole polynomial by 4:

4(x-4)(x+6)

Notice if I still plug in 4 or -6 for x, the whole thing is going to be 0.

In standard form, the polynomial would be 4x^2+8x-96.

if 26 articles cost $214.50 what amount does what amount does one article called what is the cost of 15 articles​

Answers

Answer: one costs 8.25 and 15 costs 123.75

Step-by-step explanation: 214.5/26 = 8.25 x 15 = 123.75

Answer:

15 articles cost $123.75

Step-by-step explanation:

In order to find the answer, you first need to divide $214.50 by 26 which is $8.25 for per article, and multiply it by 15 to find how much it cost for 15 articles which is $123.75

Ricarda earns $25 allowance each week. How can she mentally compute her total earnings after 7 weeks?

Answers

Answer:

25w

Step-by-step explanation:

Let w represent the number of weeks and 25 is the amount she gets each week, so 25(7)=175.

Answer: $175

Step-by-step explanation: She could do 25x8(25x4x2) which is 200 and then subtract 25.

On Friday, a bowling alley made $842.72 from lane rentals and $412.38 from the concession stand. On Saturday, their lane rentals were down by StartFraction 1 over 8 EndFraction but the concessions increased by One-half. What is the total amount that the bowling alley earned in lane rentals and concessions on Friday and Saturday?
$1,355.95
$1,580.10
$1,992.48
$2,611.05

Answers

Total: 737.38 + 618.57 = 1,355.95 Choice A.

Answer:

D

Step-by-step explanation:

3 (3 – 3) = 2 ( + 3) – 30

Answers

Answer:

0 = 2×3 - 30

= 6-30

= -24

is the right answer

This question has no solution, but If you want to to compare, then it's False because 0 doesn't equal -24

Which of the following is true about the function above?​ - Apex

Answers

Answer:

The answer is B, The domain is (-4, infinite) and it's range is (0, infinite)

Step-by-step explanation:

If we graph this equation, we can see that x=-4 is the ascending point on the graph, infinitely continuing upwards, giving us -4, infinite. The range, however is found with infinitely continuing off the graph to the right, making it a positive infinitely for the y value as it is NOT a steady straight line.

The temperature at 5:00 a.m. was –6 °F. By 3:00 p.m., the temperature had risen to a high of 36 °F. If the temperature at 8:00 p.m. decreased from the high by latex of the change in temperature in Fahrenheit degrees from 5:00 a.m. to 3:00 p.m., what was the temperature at 8:00 p.m.?

Answers

Complete question :

The temperature at 5:00 a.m. was –6 °F. By 3:00 p.m., the temperature had risen to a high of 36 °F. If the temperature at 8:00 p.m. decreased from the high by the change in temperature in Fahrenheit degrees from 5:00 a.m. to 3:00 p.m., what was the temperature at 8:00 p.m.?

Answer:

-6°F

Step-by-step explanation:

Temperature at 5am (t1) = - 6°F

Temperature at 3pm (t2) = 36°F

Change in temperature, between 5am and 3pm:

(t2 - t1) = (36 - (-6)) = (36 + 6) = 42°F

Hence, temperature at 8:00 pm:

High temperature - change in temperature between (5am - 3pm)

36°F - 42°F = - 6°F

Hector wants to tile his rectangular shed floor with rubber tiles. He found a company that will cut custom tiles up to 60 inches by 60 inches. The length of his shed floor is 240 inches, and the width is 144 inches. A-what is the side length of the largest square tile Hector can use to tile his shed floor so that there are no gaps or overlaps and the entire floor is covered? B-How many floor tiles will Hector need to cover his shed floor? HURRY PLEASE. ANSWER BOTH!!

Answers

Answer:

A- The side length of the largest square tile Hector can use to tile his shed floor so that there are no gaps or overlaps is 48 inches

B- The number of square tiles Hector will need is 15 tiles

Step-by-step explanation:

A- The given parameters are;

The size of the tiles cut by the company he found = 60 inches × 60 inches

The length of his shed floor = 240 inches

The width of his shed floor = 144 inches

The side length of the largest square tile is given as the highest common factor, HCF of 240 and 144 given as follows;

144/240 = 3/5

Therefore, we have the highest common factor of 144 and 240 = 144/3 = 48

Therefore, the side length of the largest square tile Hector can use to tile his shed floor so that there are no gaps or overlaps = 48 inches

B- The area of his shed floor = 240 inches × 144 inches = 34560 in.²

The area of one square tile = 48 inches × 48 inches = 2,304 in.²

The number of square tiles, n, Hector will need is given as follows;

The number of square tiles Hector will need = (The area of his shed floor)/(The area of one square tile)

The number of square tiles Hector will need = n =34560 in.²/(2,304 in.²) = 15

The number of square tiles Hector will need = 15 tiles.

Other Questions
If the 22nd Amendment were in place, how would it have affected the presidency during World War II? What did salutary neglect mean for the English colonies? The colonies were tightly controlled by the British empire. The colonies were free to do what they wanted as long as England was making money. The colonies were not allowed to set up their own governments. The colonies would had to pay France and Spain for protection. 7.The following is written in expanded notation.(1 x 1,000) + (7 x 10) + (6 x 1) + (1 x 160) + (4 x 1080)Write this number in standard form. To find the quotient of 8 divided by one-third, multiply 8 byOne-eighth.One-third.3.8. Michael says that the difference between danis and rajs age is proportional to danis age. Is Michael correct ? David is newly hired medical assistant at new hope clinic. during his second day on the job, he notice one of the other medical assistants slip a pharmaceutical sample box of a prescription antacid into his lab coat pocket before leaving for the day. David in unsure of what to do . law ethic both and i need a suggesions too please a help The standard format rules for a professional letter should be followed.TrueFalse List the six shifters (NPDs) of demand imagine yourself in a frozen forest Where does the threat to our world's ecosystems and quality of life come from? *5 pointsDemographic TransitionHuman PopulationMigrationCohorts What is the measure of each angle of a regular pentagon? Plz help Find the surface area of a rectangular solid with the given dimensions: length 25 feet, width 7 feet, height 10 feet. Give your answer without units. You throw a basketball and a tennis ball across the classroom so that each ball has the exact same momentum. Explain how this is possible given that a basketball has more mass than a tennis ball. What improper fraction has the same value as 12 3/4?A) 51/4B) 48/4C) 123/4D) 12/4 My parents should buy a Toyota, because it is the safest car out on the market. According to research data in 2005, Toyota drivers had the least amount of fatalities nationally. identify the Evidence of this argument A cat chases a mouse for a distance of 9.0 m over 16 s before getting tired. What is the cats average speed? qu es fcclaprograma que promueve nuestras costumbres y tradiciones Catharine, Inc. is considering issuing additional long-term debt to finance an expansion. The company currently has $20 million in 5% debt outstanding. Its earnings after-tax (EAT) are $3.0 million, and its marginal and average tax rate is 40 percent. The company is required by the debt holders to maintain its times interest earned ratio at 3.0 or greater. How much additional 10 percent debt can Catharine, Inc issue now and maintain its times interest earned ratio at 3.0 What is the nearest latitude and longitude to Florida capital Plz helpI thought the ride would never end my eyes were crossed, and my fingers were numb. What is the best way to correct this sentence?